Last visit was: 26 Apr 2024, 16:21 It is currently 26 Apr 2024, 16:21

Close
GMAT Club Daily Prep
Thank you for using the timer - this advanced tool can estimate your performance and suggest more practice questions. We have subscribed you to Daily Prep Questions via email.

Customized
for You

we will pick new questions that match your level based on your Timer History

Track
Your Progress

every week, we’ll send you an estimated GMAT score based on your performance

Practice
Pays

we will pick new questions that match your level based on your Timer History
Not interested in getting valuable practice questions and articles delivered to your email? No problem, unsubscribe here.
Close
Request Expert Reply
Confirm Cancel
SORT BY:
Kudos
Math Expert
Joined: 02 Sep 2009
Posts: 92948
Own Kudos [?]: 619238 [2]
Given Kudos: 81609
Send PM
User avatar
VP
VP
Joined: 29 Aug 2007
Posts: 1021
Own Kudos [?]: 1726 [1]
Given Kudos: 19
Send PM
User avatar
Manager
Manager
Joined: 18 Aug 2009
Posts: 214
Own Kudos [?]: 1622 [1]
Given Kudos: 9
Send PM
User avatar
VP
VP
Joined: 29 Aug 2007
Posts: 1021
Own Kudos [?]: 1726 [1]
Given Kudos: 19
Send PM
Re: PS questions about standard deviation. [#permalink]
1
Kudos
Bunuel wrote:
Questions 1 and 9 are solved incorrectly. One of two answers for 6 is incorrect.


:shock: Are you talking :shock: seriously??? :shock:
User avatar
VP
VP
Joined: 29 Aug 2007
Posts: 1021
Own Kudos [?]: 1726 [1]
Given Kudos: 19
Send PM
Re: PS questions about standard deviation. [#permalink]
1
Bookmarks
gmattokyo wrote:
9. E is a collection of four odd integers and the greatest difference between any two integers in E is 4. The standard deviation of E must be one of how many numbers?
(A) 3
(B) 4
(C) 5
(D) 6
(E) 7
-----------------------------------

(C) 5
Not very sure of this one... would wait for an expert solution.
Given that the range is 4. So pick up any set of odd numbers (SD will be same for the scenarios, all the set of 4 odd integers with range of 4 will have 3 unique members).
Possible sets (each with a different SD):
1. [1,5,1,1]
2. [1,5,5,5]
3. [1,5,3,3]
4. [1,5,1,3]
5. [1,5,1,5]
6. [1,5,3,5]

total of 6. 1st and 2nd have the same SD. Left with 5 other cases.


1. [1,5,1,1] and 2. [1,5,5,5] have SD of 2.
3. [1,5,3,3] has a SD of 1.63.
4. [1,5,1,3] and 6. [1,5,3,5] have 1.91.
5. [1,5,1,5] has a SD of 2.31.

So there are altogather 4 different SDs.
Math Expert
Joined: 02 Sep 2009
Posts: 92948
Own Kudos [?]: 619238 [1]
Given Kudos: 81609
Send PM
Re: PS questions about standard deviation. [#permalink]
1
Kudos
Expert Reply
ykaiim wrote:
Bunuel,

I think the statement in the Q10 is incorrect. Some typo error, IMO:

10. If a certain sample of data has a mean of 20.0 and a standard deviation of 3.0, which of the following values is more than 2.5 standard deviations AWAY from the mean?
(A) 12.0
(B) 13.5
(C) 17.0
(D) 23.5
(E) 26.5
Answer: A.

Please confirm.

Bunuel wrote:
10. If a certain sample of data has a mean of 20.0 and a standard deviation of 3.0, which of the following values is more than 2.5 standard deviations from the mean?

(A) 12.0
(B) 13.5
(C) 17.0
(D) 23.5
(E) 26.5
Answer: A.


No typo there: the question is from GmatPrep but basically it's the same as you wrote.
Math Expert
Joined: 02 Sep 2009
Posts: 92948
Own Kudos [?]: 619238 [1]
Given Kudos: 81609
Send PM
Re: PS questions about standard deviation. [#permalink]
1
Kudos
Expert Reply
shrive555 wrote:
GMAT TIGER wrote:
Bunuel wrote:
3. For a certain examination, a score of 58 was 2 standard deviations below the mean, and a score of 98 was 3 standard deviations above the mean. What was the mean score for the examination?
(A) 74
(B) 76
(C) 78
(D) 80
(E) 82



x - 2sd = 58
x + 3sd = 98

SD = 8 and Mean (x) = 74 in A.


how did you solve this x - 2sd = 58 and x + 3sd = 98 ?


A score of 58 was 2 standard deviations below the mean --> 58=Mean-2*sd
A score of 98 was 3 standard deviations above the mean --> 98=Mean+3*sd

Subtract (1) from (2) --> 98-58=Mean+3*sd-(Mean-2*sd) --> 40=5sd --> sd=8 --> Mean=58+2*sd=58+16=74.


Hope it's clear.
User avatar
Intern
Intern
Joined: 13 Oct 2010
Posts: 13
Own Kudos [?]: 83 [1]
Given Kudos: 0
Send PM
Re: PS questions about standard deviation. [#permalink]
1
Kudos
Interesting Questions on Mean,Median and Mode.
(Plucked from Dr.Math) I believe these type of conceptual questions would help us all. Very fundamental but somewhat tricky for a person like me. If this is not the right place to include these type of questions or that they do not qualify the post take action accordingly. Thanks.

Q1) Find a set of five data values with modes 0 and 2, median 2, and
mean 2.



Soln:
Mode = values appearing most often,
Median = value with as many other values above as below,
Mean = average (sum of the values divided by the number of values).

There are five values. If all were different, there would be five modes, but there are only two. The two modes must appear at least twice. They cannot appear three times each, because then you would have at least six values, not five. Thus four of the values must be 0, 0, 2, and 2.

For 2 to be the median, the remaining value, call it x, must be greater than 2. If 0 < x < 2, then x would be the median, and if
x < 0, 0 would be the median.

Then the mean is (0+0+2+2+x)/5 = 2, which you can solve for x.

Q2) We are told that the median of five numbers is 5, the mode is
1, and the mean is 4. Find the five numbers?



Soln: We can start by drawing a blank for each of the values. Then we'll try to fill them in, putting them in ascending order as we go. Here are the five blanks:

__ __ __ __ __

Now, what do we know about the numbers? We know that "the median of five numbers is 5." The median is the middle number when arranged in ascending order, so let's put it there:

__ __ 5 __ __

What else do we know? We're told that "the mode is 1." That means that 1 has to appear more than any other number. Since 1 is less than 5, all 1's will have to go to the left of the 5. We know we need at least two of them (otherwise, 5 would be a mode as well), so both blanks on the left will have to be 1's. Putting them in, we have:

1 1 5 __ __

Now the last clue is, "the mean is 4." The mean is the "average" of the numbers. It is computed by taking the sum of the numbers and dividing it by the number of numbers. Algebraically, if we call our values A, B, C, D and E, we'd write:

M = (A+B+C+D+E) / 5

Since we already know the first three numbers, let's plug them in for A, B, and C. We also know that the mean is 4, so we'll plug that in too:

4 = (1+1+5+D+E) / 5

4 = (7+D+E) / 5

Now let's solve for D+E, the two numbers we don't know:

4 = (7+D+E) / 5

4 * 5 = (7+D+E)

20 = 7+D+E

20 - 7 = D+E

D+E = 13

So the sum of the last two numbers must be 13. We also know that each of those two numbers must be greater than 5. What two numbers will work? Only 6 and 7 (can you think of WHY only 6 and 7 work?) So our five numbers must be:

1 1 5 6 7

To check; median = 5 (check), mode = 1 (check), mean is:

M = (1+1+5+6+7) / 5

= 20 / 5

= 4 (check)
avatar
Manager
Manager
Joined: 26 Dec 2011
Posts: 78
Own Kudos [?]: 138 [1]
Given Kudos: 17
Send PM
Re: PS questions about standard deviation. [#permalink]
1
Kudos
let me explain, it is 0,s-t,s-r, thus, ... if say we multiply this set by -....0,t-s,r-s....now add s in all the terms <adding/subtracting same number in the set does not change the SD>....the set becomes <s,t,r>...

for the third one, the set may |r|,|s|,|t|..now since...now since SD=sqrt {summation(x-mean)2}/n...u can see that SD will differ as x will differ...

I hope it is clear...

Pavan

I hope it clear.
User avatar
Manager
Manager
Joined: 28 Apr 2012
Posts: 239
Own Kudos [?]: 949 [1]
Given Kudos: 142
Location: India
Concentration: Finance, Technology
GMAT 1: 650 Q48 V31
GMAT 2: 770 Q50 V47
WE:Information Technology (Computer Software)
Send PM
Re: PS questions about standard deviation. [#permalink]
1
Kudos
Bunuel wrote:
8. The table below represents three sets of numbers with their respective medians, means and standard deviations. The third set, Set [A+B], denotes the set that is formed by combining Set A and Set B.

Median Mean StandardDeviation
Set A: X, Y, Z.
Set B: L, M, N.
Set [A + B]: Q, R, S.
If X – Y > 0 and L – M = 0, then which of the following must be true?
I. Z > N
II. R > M
III. Q > R
(A) I only
(B) II only
(C) III only
(D) I and II only
(E) None


We have been given
Set A - Elements with Median > Mean.
Set B - Elements with Median = Mean.

I. Z > N : We cannot compare standard deviations of two alien sets - False. [ we don't know what is A, what is B and what is the relationship between them] - So I Out.
II. R > M : We are comparing mean of the combined sets with mean of set B. We have no knowledge of number of elements (cardinality) of A or B. Consider this as a case of weighted average. Unless we know the number of elements of A, number of elements of B and mean values of Both the sets, we can not evaluate this.
For Instance, If A has 4 elements and mean of 5, B has 10 elements and mean of 8 .Then M =8, R = (4*5 + 10*8)/(4+10) = 100/14 = approx 7. So R < M
now, suppose Set A has a mean of 20 instead of 5. M remains unchanged. R = (4*20 + 10*8)/(4+10) = 160/14 = approx. 11. So R > M
So II is Out.
III. Q > R : Mean > Median. Again as above, since we do not know anything about composition of A and B. This might be true or false depending on distribution of A and B.
As there is no co-relationship provided between A and B, we cannot derive any conclusion.

So the answer in E. None.

The problem could be twisted, if instead of L-M=0, we would be given N=0.
In that case, Z is always >0. So answer would be A then.
User avatar
Manager
Manager
Joined: 16 Dec 2011
Posts: 236
Own Kudos [?]: 788 [1]
Given Kudos: 70
Re: PS questions about standard deviation. [#permalink]
1
Bookmarks
7. Which of the following data sets has the third largest standard deviation?
(A) {1, 2, 3, 4, 5}
(B) {2, 3, 3, 3, 4}
(C) {2, 2, 2, 4, 5}
(D) {0, 2, 3, 4, 6}
(E) {-1, 1, 3, 5, 7}


(A) standard deviation = 1.58113883
(B) standard deviation = 0.707106781
(C) standard deviation = 1.414213562
(D) standard deviation = 2.236067977
(E) standard deviation = 3.16227766

Answer is A.

To go by logic:

All the sets got same number of elements; so, not considering any effect from that.

Set E has highest range and elements are more distributed towards edge. --> set has the highest standard deviation.
Set D has next highest range and but little concentric towards mean. Still the distribution looks better than remaining sets. Thus, this set has the second highest standard deviation.
Set B has lowest range and elements are mainly centered towards mean. Thus, this set has the lowest standard deviation.
Now we are left with options A and C. Range of set A is more than range of set C. Elements of A are evenly distributed (more spread) and elements of C are concentrated towards one side. From these two observations, we can speculate that option A has higher standard deviation than that of option C.

Note: All these sets have small sample size, very small range, and the numbers are very close to each other. This scenario say little without calculation and, hopefully, real GMAT will not come up with this sort of questions.
Math Expert
Joined: 02 Sep 2009
Posts: 92948
Own Kudos [?]: 619238 [1]
Given Kudos: 81609
Send PM
Re: PS questions about standard deviation. [#permalink]
1
Kudos
Expert Reply
doe007 wrote:
Bunuel wrote:
I guess this is not real GMAT question as to answer this question with 100% certainty you should calculate SD for two sets and GMAT usually do not require actual calculation of SD. Though it's possible to eliminate 3 wrong answers at the beginning.

Mean is 4 and so are the means of all 5 pairs from answers choices.

A. (-1, 9) These two numbers are farthest from the mean and they will stretch the set making SD bigger

B. (4, 4) These two numbers are closest to the mean and the will shrink the set making SD smaller

C. (3, 5) Suitable option so far

D. (2, 6) Suitable option so far

E. (0, 8) These two numbers are also far from mean and they will also stretch the set making SD bigger.

So, when I looked at the options C and D I assumed that C is also too close to the mean and it will affect it more than D. So I ended with D and was correct. But still my logic eliminating C was not sure thing, without the calculations.


For the original set 0, 2, 4, 6, 8, standard deviation = 3.16227766

A) For the set -1, 0, 2, 4, 6, 8, 9, standard deviation = 3.872983346
B) For the set 0, 2, 4, 4, 4, 6, 8, standard deviation = 2.581988897
C) For the set 0, 2, 3, 4, 5, 6, 8, standard deviation = 2.645751311
D) For the set 0, 2, 2, 4, 6, 6, 8, standard deviation = 2.828427125
E) For the set 0, 0, 2, 4, 6, 8, 8, standard deviation = 3.464101615


[As the sample size is very small, SD's are calculated using formula for sample.]

Difference between original stdev and stdev of option D is 0.333850535
Difference between original stdev and stdev of option E is 0.301823955

Though option D has a very close call, closest to the original standard deviation is found in option E.

Correct answer is option E.

I believe this is not a real GMAT question and GMAC would not ask for such lengthy calculations.

Bunuel, you said correctly that answer to this question can be found through calculation only. I am always impressed by your explanations. For this question, however, something is amiss. I wish there is a simple way to find correct answer for this question.


That's not correct.

SD of {0, 2, 4, 6, 8} = ~2.83 (3.16 is sample standard deviation and 2.83 is population standard deviation, which is tested on the GMAT).

C. SD of {0, 2, 4, 6, 8, 3, 5} = ~2.45 --> difference=0.38
D. SD of {0, 2, 4, 6, 8, 2, 6} = ~2.62 --> difference=0.21
E. SD of {0, 2, 4, 6, 8, 0, 8} = ~3.21 --> difference=0.38

The correct answer is D.
Current Student
Joined: 14 Nov 2016
Posts: 1174
Own Kudos [?]: 20719 [1]
Given Kudos: 926
Location: Malaysia
Concentration: General Management, Strategy
GMAT 1: 750 Q51 V40 (Online)
GPA: 3.53
Send PM
Re: PS questions about standard deviation. [#permalink]
1
Bookmarks
IanStewart wrote:
Bunuel wrote:



8. The table below represents three sets of numbers with their respective medians, means and standard deviations. The third set, Set [A+B], denotes the set that is formed by combining Set A and Set B.

Median Mean StandardDeviation
Set A: X, Y, Z.
Set B: L, M, N.
Set [A + B]: Q, R, S.
If X – Y > 0 and L – M = 0, then which of the following must be true?
I. Z > N
II. R > M
III. Q > R
(A) I only
(B) II only
(C) III only
(D) I and II only
(E) None


We have no information that might allow us to compare Z and N, so I need not be true. For II, without knowing the relationship between Y and M, we cannot decide whether R is larger than M. For III, if set A is {0, 3, 4}, then the median of A is larger than the mean. If set B is {13}, then the median of B is equal to the mean. So these sets agree with the conditions given. Combining the sets, we have {0, 3, 4, 13}, which has a median of 3.5 and a mean of 5; the median is not greater than the mean. So III need not be true and the answer is E.


Official answer from Manhattan Prep.

If X – Y > 0, then X > Y and the median of A is greater than the mean of set A. If L – M = 0, then L = M and the median of set B is equal to the mean of set B.

I. NOT NECESSARILY: According to the table, Z > N means that the standard deviation of set A is greater than that of set B. Standard deviation is a measure of how close the terms of a given set are to the mean of the set. If a set has a high standard deviation, its terms are relatively far from the mean. If a set has a low standard deviation, its terms are relatively close to the mean.

Recall that a median separates the set into two as far as the number of terms. There is an equal number of terms both above and below the median. If the median of a set is greater than the mean, however, the terms below the median must collectively be farther from the median than the terms above the median. For example, in the set {1, 89, 90}, the median is 89 and the mean is 60. The median is much greater than the mean because 1 is much farther from 89 than 90 is.

Knowing that the median of set A is greater than the mean of set A just tells us that the terms below set A’s median are further from the median than the terms above set A’s median. This does not necessarily imply that the terms, overall, are further away from the mean than in set B, where the terms below the median are the same distance from the median as the terms above it. In fact, a set in which the mean and median are equal can have a very high standard deviation if the terms are both far below the mean and far above it.

II. NOT NECESSARILY: According to the table, R > M implies that the mean of set [A + B] is greater than the mean of set B. This is not necessarily true. When two sets are combined to form a composite set, the mean of the composite set must either be between the means of the individual sets or be equal to the mean of both of the individual sets. To prove this, consider the simple example of one member sets: A = [3], B = [5], A + B = [3, 5]. In this case the mean of A + B is greater than the mean of A and less than the mean of B. We could easily have reversed this result by reversing the members of sets A and B.

III. NOT NECESSARILY: According to the table, Q > R implies that the median of the set [A + B] is greater than the mean of set [A + B]. We can extend the rule given in statement II to medians as well: when two sets are combined to form a composite set, the median of the composite set must either be between the medians of the individual sets or be equal to the median of one or both of the individual sets. While the median of set A is greater than the mean of set A and the median of set B is equal to the mean of set B, set [A + B] might have a median that is greater or less than the mean of set [A + B].

Therefore none of the statements are necessarily true and the correct answer is E.
Attachments

Untitled.jpg
Untitled.jpg [ 43.94 KiB | Viewed 7113 times ]

User avatar
Intern
Intern
Joined: 02 Oct 2009
Posts: 15
Own Kudos [?]: 203 [0]
Given Kudos: 5
Send PM
Re: PS questions about standard deviation. [#permalink]
Bunuel wrote:

6. A certain characteristic in a large population has a distribution that is symmetric about the mean m. If 68% of the distribution lies one standard deviation d of the mean, what percent of the distribution is less than m+d?
(A) 16%
(B) 32%
(C) 48%
(D) 84%
(E) 92%



I m a little confused between 32% and 84% but i think 84% makes more sense.

If it is symmetric about the mean, then data less than m is and data greater than m is both 50%.

Then, each of the area from m to m+d and m to m-d would be 68/2=34%.

Thus, for data to be less than m+d, we have

Data from m+d to m = 34%
Data less than m = 50%
Thus, total 84% (D)
User avatar
Manager
Manager
Joined: 18 Aug 2009
Posts: 214
Own Kudos [?]: 1622 [0]
Given Kudos: 9
Send PM
Re: PS questions about standard deviation. [#permalink]
Bunuel wrote:
Questions 1 and 9 are solved incorrectly. One of two answers for 6 is incorrect. 10 and 11 aren't solved yet, though they are relatively easy.

The hardest questions in this set are 8 and 9. Probably they are 750+ problems, so would be interesting to see the solutions for them. Also please note that I don't have the OA for 8!, only my own solution.

Good luck.


I'll attempt an easy one :oops:

10) A
Mean = 20.0
S.D = 3
More than 2.5 SD from mean means either less than 20- (2.5x3=7.5) = 12.5 or greater than 20+7.5= 27.5
User avatar
Manager
Manager
Joined: 18 Aug 2009
Posts: 214
Own Kudos [?]: 1622 [0]
Given Kudos: 9
Send PM
Re: PS questions about standard deviation. [#permalink]
gmattokyo wrote:
Bunuel wrote:
Questions 1 and 9 are solved incorrectly. One of two answers for 6 is incorrect. 10 and 11 aren't solved yet, though they are relatively easy.

The hardest questions in this set are 8 and 9. Probably they are 750+ problems, so would be interesting to see the solutions for them. Also please note that I don't have the OA for 8!, only my own solution.

Good luck.


I'll attempt an easy one :oops:

10) A
Mean = 20.0
S.D = 3
More than 2.5 SD from mean means either less than 20- (2.5x3=7.5) = 12.5 or greater than 20+7.5= 27.5


By The same logic
11) A
13.5-3=10.5
User avatar
Manager
Manager
Joined: 18 Aug 2009
Posts: 214
Own Kudos [?]: 1622 [0]
Given Kudos: 9
Send PM
Re: PS questions about standard deviation. [#permalink]
Trying the difficult one...

8. The table below represents three sets of numbers with their respective medians, means and standard deviations. The third set, Set [A+B], denotes the set that is formed by combining Set A and Set B.

Median Mean StandardDeviation
Set A: X, Y, Z.
Set B: L, M, N.
Set [A + B]: Q, R, S.
If X – Y > 0 and L – M = 0, then which of the following must be true?
I. Z > N
II. R > M
III. Q > R
(A) I only
(B) II only
(C) III only
(D) I and II only
(E) None

----------------------------------
(C) III only
I. SD of one set is greater than another. We cannot prove this as no information is given on the actual data of the individual sets
II. Mean of combined set may or may not be greater than one of the given sets.
Consider Set A [1, 3, 3, 3] Median-3, Mean-2.5
Set B [1, 4, 4, 4] Median-4, Mean-3.25
Set A+B [1, 1, 3, 3, 3, 4, 4, 4] Median-3, Mean-2.8
In this case R is not greater than M. But if you interchange set A & B, R>M.
Math Expert
Joined: 02 Sep 2009
Posts: 92948
Own Kudos [?]: 619238 [0]
Given Kudos: 81609
Send PM
Re: PS questions about standard deviation. [#permalink]
Expert Reply
GMAT TIGER wrote:
Bunuel wrote:
Questions #1 and #9 are solved incorrectly. One of two answers for #6 is incorrect.


:shock: Are you talking :shock: seriously??? :shock:


Yes. Your second solution for question #9 is correct. Indeed there are 6 possibilities and 2 pairs have the same SD, so there would be 4 different SD.

As for #1: if it would be E {0,8}, wouldn't these two numbers stretch the set making SD bigger? As {0,8} are too far from the the mean.

And for #6: 2 answers were given (by others) to this question and one of them is incorrect.

So yes, I'm serious. But maybe I didn't get your question right...
User avatar
VP
VP
Joined: 29 Aug 2007
Posts: 1021
Own Kudos [?]: 1726 [0]
Given Kudos: 19
Send PM
Re: PS questions about standard deviation. [#permalink]
Bunuel wrote:
GMAT TIGER wrote:
Bunuel wrote:
Questions #1 and #9 are solved incorrectly. One of two answers for #6 is incorrect.


:shock: Are you talking :shock: seriously??? :shock:


Yes. Your second solution for question #9 is correct. Indeed there are 6 possibilities and 2 pairs have the same SD, so there would be 4 different SD.

As for #1: if it would be E {0,8}, wouldn't these two numbers stretch the set making SD bigger? As {0,8} are too far from the the mean.


Agree with your logic but did you check whether other choices could deviate more further than E?
So what is your OA and workout for #1.
Did you solve it or just guessing? If you solve it, you will find E as the closest.
Math Expert
Joined: 02 Sep 2009
Posts: 92948
Own Kudos [?]: 619238 [0]
Given Kudos: 81609
Send PM
Re: PS questions about standard deviation. [#permalink]
Expert Reply
GMAT TIGER wrote:
Agree with your logic but did you check whether other choices could deviate more further than E?
So what is your OA and workout for #1.
Did you solve it or just guessing? If you solve it, you will find E as the closest.


Frankly speaking when I solved this for the first time I didn't calculated SD-s. Just trusted my logic, then compared my answer to the OA and as they matched I didn't double checked my own solution and OA.

But know I did it. And I can say it again E {0,8} is not correct.

Maybe there is some misunderstanding in stem?
GMAT Club Bot
Re: PS questions about standard deviation. [#permalink]
   1   2   3   
Moderators:
Math Expert
92948 posts
Senior Moderator - Masters Forum
3137 posts

Powered by phpBB © phpBB Group | Emoji artwork provided by EmojiOne